Estabilidade de Routh-Hurwitz

download Estabilidade de Routh-Hurwitz

of 15

Transcript of Estabilidade de Routh-Hurwitz

  • 03/05/2011

    1

    6.2. O critrio de Estabilidade de Routh-Hurwitz

    A. Hurwitz e E.J. Routh publicaram independentemente um mtodo de investigar a

    estabilidade de um sistema linear (vide Ogata).

    O critrio de estabilidade de Routh-Hurwitz verifica se todos os plos de uma funo de

    transferncia pertence ao semi-plano esquerdo do plano-s.

    Suponha que a funo de transferncia da forma:

    00

    1

    1

    10

    1

    1

    10

    ++++

    ++++=

    aasasasa

    bsbsbsbsG

    nnnn

    mmmm

    ,...

    ...)(

    1passo: identifique apenas o denominador de G(s):

    nnnn asasasasD ++++=

    1

    1

    10...)(

    2passo: verifique se qualquer destas constantes (ai) igual a zero ou, negativa na

    presena de pela menos uma constante positiva. Se isto ocorrer, conclua que o

    sistema instvel e no necessrio executar os prximos passos.

    Do contrrio, nada pode-se concluir, v para o 3 passo.

    (1)

    1

    3passo: construa a seguinte tabela:

    Os elementos a0, a1, ...,an so os coeficientes do denominador D(s) da equao (1).

    2

  • 03/05/2011

    2

    Os elementos b1, b2, b3, ..., c1, c2, ... e todos os demais so calculados com as

    seguintes expresses:

    3

    4passo: aplique o seguinte critrio de estabilidade de Routh-Hurwitz:

    O nmero de razes de D(s) (plos de G(s)) com parte real maior que zero (positivo)

    igual ao nmero de mudanas de sinal dos coeficientes da primeira coluna da

    tabela construda no 3passo.

    Obs.: se pelo menos um elemento da 1. coluna for nulo ou se uma linha toda for

    nula, deve-se observar o caso especial que mostraremos mais adiante.

    Exemplo: Seja estude sua estabilidade.5432

    12

    234++++

    +=

    ssss

    ssG )(

    Sol.:

    1 passo: 5432234

    ++++= sssssD )(

    2passo: todos coeficientes de D(s) so positivos portanto nada pode-se concluir.

    4

  • 03/05/2011

    3

    3passo:

    Neste caso, os elementos da 1coluna so:

    5

    Exemplo: Determine se o sistema estvel ou instvel:

    53

    1

    23

    2

    ++

    +=

    sss

    sssG )(

    Sol.: 1 passo: 5323

    ++= ssssD )(

    2passo: existe um coeficiente negativo na presena de outro positivo, logo

    o sistema instvel e no precisa ir ao passo seguinte.

    Exerccio: O piloto automtico de um avio tem a seguinte F.T.M.F.:

    verifique se o sistema estvel ou instvel.

    9244166761303524015

    900165900150

    2345

    23

    +++++

    +++=

    sssss

    sss

    s

    s

    r ,,,)(

    )(

    >>den=[1 3 -1 5];

    >>roots(den) 6

  • 03/05/2011

    4

    >>den=[1 3 -1 5];

    >>roots(den)

    Aparentemente o mtodo de Routh-Hurwitz seria desnecessrio, porm ele

    extramente til para projetar controladores, o exemplo a seguir ilustra este fato.

    O clculo dos plos de um sistema (razes de um polinmio) so fceis para os usurios

    do MATLAB ou das calculadoras cientficas atuais. Por exemplo, os plos do exemplo

    acima so calculados pelo MATLAB com o comando:

    Exemplo: Determine o intervalo de k, ganho do controlador, para o qual o sistema

    realimentado seja estvel.

    7

    sol.: A F.T.M.F. dada por:

    )())((

    )(

    ))((

    )())((

    )(

    )(161

    1

    61

    11

    61

    1

    +++

    +=

    +

    ++

    +

    +

    =sksss

    sk

    sss

    sk

    sss

    sk

    sH

    Note que no possvel obter os plos de H(s) usando a calculadora.

    Usemos o mtodo de Routh-Hurwitz:

    1passo: D(s)=s3+5s2+(k-6)s+k

    2passo: Para que todos os coeficientes sejam positivos:

    k-6>0 k>6

    e

    k>0

    k>6 satisfaz (I)

    8

  • 03/05/2011

    5

    3passo:

    Para que elementos da 1 coluna sejam todos positivos, necessrio que:

    574

    30030403050

    5

    65,

    )(>>>>>

    kkkkk

    kk (II)

    e

    k>0 (III)

    Logo, para k>7,5 o sistema ser estvel.

    Como j foi dito, se tiver um zero (0) na primeira coluna de tabela ou se

    uma linha for nula, ento deve-se usar o caso especial a seguir. 9

    Exerccio: Estude a estabilidade do sistema.

    Exerccio: Estude a estabilidade do sistema.

    10

  • 03/05/2011

    6

    CASO ESPECIAL

    Se o primeiro elemento de uma linha zero, e pelo menos um elemento na

    mesma linha diferente de zero, ento substituiu-se o primeiro elemento de

    linha, que zero, por um pequeno nmero , que poder ser negativo ou

    positivo, e continua-se o clculo das prximas linhas da tabela. O exemplo

    abaixo ilustra este caso.

    Exemplo: Estude a estabilidade de

    1011422

    5

    2345+++++

    =sssss

    sG )(

    1passo: D(s)=s5+2s4+2s3+4s2+11s+10

    Sol.:

    2passo: todos os coeficientes so positivos, nada pode-se concluir.

    3passo: construo da tabela:

    11

    neste caso aparece um 0 na 1 coluna e outros elementos

    desta linha so diferentes de 0. Mostre que no possvel

    calcular os elementos da linha s2 pois seria necessrio

    dividir por 0. Substitua o 0 por e continue:

    12

  • 03/05/2011

    7

    para pequeno, 0, tem-se a seguinte tabela:

    se 0 temos 6

    13

    Se 0 pela esquerda, ou seja 0, temos tambm 2 trocas de sinais na

    primeira coluna.

    Assim, o sistema instvel.

    14

  • 03/05/2011

    8

    Exerccio: Estude a estabilidade de:

    96623

    7

    2345+++++

    =sssss

    SG )(

    Exerccio: Encontre a faixa de K tal que o sistema abaixo seja estvel:

    Exerccio: Determinar a faixa de valores de K para a qual o sistema a seguir

    estvel:

    15

    Estabilidade de sistema com projeto de controlador dependente de dois

    parmetros

    Um controlador industrial muito utilizado o controlador P.I. (proporcional e

    integral).

    Neste caso a estabilidade fica dependente de dois parmetros. Um exemplo

    de projeto ilustra o uso do critrio de estabilidade de Routh-Hurwitz para este

    caso, e est mostrado a seguir.

    Exemplo: Para o sistema controlado por um controlador P.I. dado abaixo,

    encontre as faixas de kp e ki do controlador tal que o sistema abaixo seja

    estvel:

    16

  • 03/05/2011

    9

    Sol.: A F.T.M.F.

    ip

    ip

    ip

    ip

    ksksss

    ksk

    sss

    ksksss

    ksk

    sR

    sY

    ++++

    +=

    ++

    ++

    ++

    +

    =))((

    ))((.)(

    ))((.

    )(

    )(

    21

    21

    11

    21

    1

    ip

    ip

    kskss

    ksk

    sR

    sY

    ++++

    +=

    )()(

    )(

    2323

    1passo: D(s)=s3+3s2+(2+kp)s+ki

    2passo: para estabilidade necessrio que:

    0>ike: 202 >>+ pp kk (I)

    17

    3passo:

    1 coluna:

    23>

    ip

    kk (II)

    (III)e 0>ik

    023 >+ ip kk )(

    De (I), (II) e (III) tem-se a regio:

    18

  • 03/05/2011

    10

    Exerccio: Encontre a faixa de kp e ki do controlador abaixo tal que o sistema

    seja estvel.

    19

    Matlab (Simulink):

    6.3. Estabilidade Relativa

    A estabilidade estudada at agora neste curso conhecida como

    estabilidade absoluta pois tem-se como referncia o lado esquerdo do plano-

    s. Um outro conceito o conceito de estabilidade relativa.

    Pode-se determinar a margem de segurana que um sistema apresenta no

    tocante sua estabilidade. Por exemplo, no plano-s abaixo, pode-se dizer

    que os plos z1 e z1 tem menor margem de estabilidade que os plos z2 e z3:

    20

  • 03/05/2011

    11

    Pode-se usar o critrio de Routh para estudar a margem de estabilidade

    relativa de um sistema, neste caso necessrio usar uma translao de

    eixo imaginrio.

    Os eixos acima so relacionados atravs da

    seguinte equao de translao de eixos:

    ou ainda+= ss '

    = 'ss

    21

    Exemplo: Verifique se o sistema abaixo tem todos os plos esquerda de s=-1:

    24269

    1

    23+++

    =sss

    sG )(

    Sol.: Neste caso, deve-se realizar a translao de eixos abaixo:

    logo, s=s-1 em G(s):

    A translao do eixo imaginrio feita substituindo s=s-1 em G(s):

    24126191

    1

    23+++

    =)'()'()'(

    )'(sss

    sG

    22

  • 03/05/2011

    12

    242626129121

    1

    22+++++

    =')''()'')('(

    )'(ssssss

    sG

    ento,

    6116

    1

    23+++

    ='''

    )'(sss

    sG

    logo,

    este sistema estvel, sua estabilidade relativa engloba o eixo s=-1.

    Portanto sua margem de estabilidade >1.

    Obs.: Para determinar a margem de estabilidade (total) de um sistema

    necessrio ir transladando o eixo s (imaginrio) at o surgimento de um zero

    na 1 coluna da tabela de Routh-Hurwitz, indicando que existe plo sobre o

    eixo imaginrio s. 23

    Este trabalho pode ser evitado, utilizando-se as calculadoras cientficas para

    obter todos os plos do sistema (ou o MATLAB), a margem de estabilidade

    ser igual ao mdulo da parte real do plo mais prximo ao eixo imaginrio,

    supondo-se que todos os plos so de sistema estvel.

    Exerccio: Use o MATLAB/SciLab ou a calculadora para determinar a

    margem de estabilidade do sistema.

    46423

    +++=

    sss

    ssG )(

    Exerccio: Verifique, usando o critrio de Routh-Hurwitz se o sistema abaixo

    tem todos seus plos esquerda de s=-2.

    423

    10

    234++++

    +=

    ssss

    ssG

    ,)(

    Exerccio: Projete K tal que o sistema abaixo tenha margem de estabilidade

    maior que 4.

    24

  • 03/05/2011

    13

    25

    Exerccio: Use o MATLAB/SciLab ou a calculadora para determinar a

    margem de estabilidade do sistema.

    46423

    +++=

    sss

    ssG )(

    Soluo:

    -->s=poly(0,"s")

    s =

    s

    -->p=s^3+4*s^2+6*s+4

    p =

    2 3

    4 + 6s + 4s + s

    -->roots(p)

    ans =

    - 1. + i

    - 1. - i

    - 2.

    -->s=poly(1,"s")

    s =

    - 1 + s

    -->p=s^3+4*s^2+6*s+4

    p =

    2 3

    1 + s + s + s

    -->roots(p)

    ans =

    - 1.145D-17 + i

    - 1.145D-17 - i

    - 1.

    -->s=poly(2,"s")

    s =

    - 2 + s

    -->p=s^3+4*s^2+6*s+4

    p =

    2 3

    2s - 2s + s

    -->roots(p)

    ans =

    0

    1. + i

    1. - i

    -->n=s

    n =

    - 2 + s

    -->d=p

    d =

    2 3

    2s - 2s + s

    -->h=syslin('c',n./d);

    -->plzr(h)

    No SciLab:

    26

  • 03/05/2011

    14

    27

    4122232

    102

    234++++

    +=

    )'()'()'()'(

    ,)'()'(

    ssss

    ssG

    4275

    91

    234++

    =

    ''''

    ,')'(

    ssss

    ssG

    Usemos o mtodo de Routh-Hurwitz:

    4275234

    ++= '''')'( sssssD2passo: existe um coeficiente negativo na presena de outro positivo, logo

    o sistema instvel e no precisa ir ao passo seguinte.

    Exerccio: Verifique, usando o critrio de Routh-Hurwitz se o sistema abaixo

    tem todos seus plos esquerda de s=-2.

    423

    10

    234++++

    +=

    ssss

    ssG

    ,)(

    Soluo:

    1passo:

    28

    -->s=poly(2,"s")

    s =

    - 2 + s

    -->n=s+0.1

    n =

    - 1.9 + s

    -->d=s^4+3*s^3+s^2+2*s+4

    d =

    2 3 4

    - 4 + 2s + 7s - 5s + s

    -->real(roots(d))

    ans =

    0.8124973

    - 0.7009008

    2.4442018

    2.4442018

    -->h=syslin('c',n./d);

    -->plzr(h)

    No SciLab:

  • 03/05/2011

    15

    29

    Exerccio: Projete K tal que o sistema abaixo tenha margem de estabilidade

    maior que 4.

    Soluo: A F.T.M.F. dada por:

    Kss

    K

    ss

    Kssss

    K

    ssK

    ssK

    sH+++

    =

    ++

    +++

    ++=

    +++

    ++=

    4022

    4022

    4022

    4022

    4022

    11

    4022

    1

    2

    2

    2

    2

    2

    2

    .

    .)(

    Para que o sistema tenha margem de estabilidade maior que 4, s=-5, temos:

    Kss

    K

    Kss

    KsH

    ++=

    +++=

    451240522522 '')'()'(

    )(

    Usando o mtodo de Routh-Hurwitz:

    1passo: KsssD ++= 45122 '')(

    30

    3passo: As razes de D(s) so dadas abaixo, para K=46, pelo SciLab:

    2passo: para estabilidade necessrio que:

    045 >+ K 45> K

    -->s=poly(0,"s")

    s =

    s

    -->K=46

    K =

    46.

    -->p=s^2+12*s+45+K

    p =

    2

    1 + 12s + s

    -->roots(p)

    ans =

    - 0.0839202

    - 11.91608

    -->n=K

    n =

    46.

    -->d=p

    d =

    2

    1 + 12s + s

    -->h=syslin('c',n./d);

    -->plzr(h)

    No SciLab: